K
Khách

Hãy nhập câu hỏi của bạn vào đây, nếu là tài khoản VIP, bạn sẽ được ưu tiên trả lời.

2 tháng 10 2021

Tham khảo:

Với các số thực không âm a,b,c thỏa mãn \(a^2+b^2+c^2=1\), tìm giá trị lớn nhất, giá trị nhỏ nhất của biểu thức:  \(Q=\s... - Hoc24

NV
30 tháng 8 2021

\(a^2+ab+b^2=\dfrac{1}{2}\left(a^2+b^2\right)+\dfrac{1}{2}\left(a+b\right)^2\ge\dfrac{1}{4}\left(a+b\right)^2+\dfrac{1}{2}\left(a+b\right)^2=\dfrac{3}{4}\left(a+b\right)^2\)

\(\Rightarrow\sqrt{a^2+ab+b^2}\ge\sqrt{\dfrac{3}{4}\left(a+b\right)^2}=\dfrac{\sqrt{3}}{2}\left(a+b\right)\)

Tương tự và cộng lại:

\(P\ge\sqrt{3}\left(a+b+c\right)=\sqrt{3}\)

\(P_{min}=\sqrt{3}\) khi \(a=b=c=\dfrac{1}{3}\)

9 tháng 12 2018

2) \(S=a+\frac{1}{a}=\frac{15a}{16}+\left(\frac{a}{16}+\frac{1}{a}\right)\)

Áp dụng BĐT AM-GM ta có:

\(S\ge\frac{15a}{16}+2.\sqrt{\frac{a}{16}.\frac{1}{a}}=\frac{15.4}{16}+2.\sqrt{\frac{1}{16}}=\frac{15}{4}+2.\frac{1}{4}=\frac{15}{4}+\frac{1}{2}=\frac{15}{4}+\frac{2}{4}=\frac{17}{4}\)

\(S=\frac{17}{4}\Leftrightarrow a=4\)

Vậy \(S_{min}=\frac{17}{4}\Leftrightarrow a=4\)

9 tháng 12 2018

kudo shinichi sao cách làm giống của thầy Hồng Trí Quang vậy bạn?

\(S=a+\frac{1}{a}=\frac{15}{16}a+\left(\frac{a}{16}+\frac{1}{a}\right)\ge\frac{15}{16}a+2\sqrt{\frac{1.a}{16.a}}=\frac{15}{16}a+2.\frac{1}{4}\)

\(=\frac{15}{16}.4+\frac{1}{2}=\frac{17}{4}\Leftrightarrow a=4\)

Dấu "=" xảy ra khi a = 4

Vậy \(S_{min}=\frac{17}{4}\Leftrightarrow a=4\)

1 tháng 2 2022

đề sai

1 tháng 2 2022

sai là sai thế nào

NV
12 tháng 11 2021

Em tham khảo ở đây:

xét các số thực a,b,c (a≠0) sao cho phương trình ax2+bx+c=0 có 2 nghiệm m, n thỏa mãn \(0\le m\le1;0\le m\le1\). tìm GTN... - Hoc24

12 tháng 11 2021

vậy không có tìm GTLN hay sao ạ?

Cần các cao nhân giải khác phương pháp SS

Không làm theo cách đánh giá 3(a2b+b2c+c2a)\(\le\)(a+b+c)(a2+b2+c2)=3(a2+b2+c2)

Ai làm được xin cảm ơn trước

22 tháng 7 2019

#)Giải :

Ta có : \(3\left(a^2+b^2+c^2\right)=\left(a+b+c\right)\left(a^2+b^2+c^2\right)\)

\(=a^3+b^3+c^3+a^2b+b^2c+c^2a+ab^2+bc^2+ca^2\)

Áp dụng BĐT Cauchy :

\(\hept{\begin{cases}a^3+ab^2\ge2a^2b\\b^3+bc^2\ge2b^2c\\c^3+ca^2\ge2c^2a\end{cases}}\)

\(\Rightarrow P\ge a^2+b^2+c^2+\frac{ab+bc+ca}{a^2+b^2+c^2}\)

\(\Rightarrow P\ge a^2+b^2+c^2+\frac{9-\left(a^2+b^2+c^2\right)}{2\left(a^2+b^2+c^2\right)}\)

Đặt \(t=a^2+b^2+c^2\Rightarrow t\ge3\)

\(\Rightarrow P\ge t+\frac{9-t}{2t}=\frac{t}{2}+\frac{9}{2t}+\frac{t}{2}-\frac{1}{2}\ge3+\frac{3}{2}-\frac{1}{2}=4\)

\(\Rightarrow P\ge4\Rightarrow P_{min}=4\)

Dấu ''='' xảy ra khi a = b = c = 1

NV
24 tháng 8 2021

\(\dfrac{1}{\left(a+b+a+c\right)^2}\le\dfrac{1}{4\left(a+b\right)\left(a+c\right)}=\dfrac{1}{4\left(a^2+ab+bc+ca\right)}\le\dfrac{1}{64}\left(\dfrac{1}{a^2}+\dfrac{1}{ab}+\dfrac{1}{bc}+\dfrac{1}{ca}\right)\)

\(\le\dfrac{1}{64}\left(\dfrac{1}{a^2}+\dfrac{1}{a^2}+\dfrac{1}{b^2}+\dfrac{1}{c^2}\right)=\dfrac{1}{64}\left(\dfrac{2}{a^2}+\dfrac{1}{b^2}+\dfrac{1}{c^2}\right)\)

Tương tự và cộng lại:

\(P\le\dfrac{1}{64}\left(\dfrac{4}{a^2}+\dfrac{4}{b^2}+\dfrac{4}{c^2}\right)=\dfrac{1}{16}.3=\dfrac{3}{16}\)

Dấu "=" xảy ra khi \(a=b=c=1\)

 

24 tháng 8 2021

Áp dụng bđt: \(\dfrac{1}{x+y}\le\dfrac{1}{4}\left(\dfrac{1}{x}+\dfrac{1}{y}\right)\left(1\right)\)

\(\dfrac{1}{2a+b+c}=\dfrac{1}{\left(a+b\right)+\left(a+c\right)}\le\dfrac{1}{4}\left(\dfrac{1}{a+b}+\dfrac{1}{a+c}\right)\)

\(\Rightarrow P\le\dfrac{1}{16}\left[\left(\dfrac{1}{a+b}+\dfrac{1}{a+c}\right)^2+\left(\dfrac{1}{a+b}+\dfrac{1}{b+c}\right)^2+\left(\dfrac{1}{b+c}+\dfrac{1}{a+c}\right)^2\right]\)\(\Rightarrow16P\le\dfrac{2}{\left(a+b\right)^2}+\dfrac{2}{\left(b+c\right)^2}+\dfrac{2}{\left(a+c\right)^2}+\dfrac{2}{\left(a+b\right)\left(b+c\right)}+\dfrac{2}{\left(a+b\right)\left(b+c\right)}+\dfrac{2}{\left(b+c\right)\left(c+a\right)}\)

Áp dụng: \(x^2+y^2+z^2\ge xy+yz+xz\left(2\right)\) với a+b=x,b+c=y,c+a=z

\(\Rightarrow16P\le\dfrac{4}{\left(a+b\right)^2}+\dfrac{4}{\left(b+c\right)^2}+\dfrac{4}{\left(c+a\right)^2}\)

Ta có: \(\dfrac{1}{\left(a+b\right)^2}\le4.16.\left(\dfrac{1}{a}+\dfrac{1}{b}\right)^2\)(do (1))

\(\Rightarrow16P\le\dfrac{1}{4}.16\left[\left(\dfrac{1}{a}+\dfrac{1}{b}\right)^2+\left(\dfrac{1}{b}+\dfrac{1}{c}\right)^2+\left(\dfrac{1}{c}+\dfrac{1}{a}\right)^2\right]=\dfrac{1}{4}\left(\dfrac{1}{a^2}+\dfrac{1}{b^2}+\dfrac{1}{c^2}+\dfrac{2}{ab}+\dfrac{2}{bc}+\dfrac{2}{ca}\right)\le\dfrac{1}{4}.4.\left(\dfrac{1}{a^2}+\dfrac{1}{b^2}+\dfrac{1}{c^2}\right)=3\)(do(2) và \(\dfrac{1}{a^2}+\dfrac{1}{b^2}+\dfrac{1}{c^2}=3\))

\(\Rightarrow P\le\dfrac{3}{16}\)

\(ĐTXR\Leftrightarrow a=b=c=1\)